4 Đề kiểm tra học kỳ I môn Tiếng Anh Lớp 12

doc 46 trang thaodu 8751
Bạn đang xem 20 trang mẫu của tài liệu "4 Đề kiểm tra học kỳ I môn Tiếng Anh Lớp 12", để tải tài liệu gốc về máy bạn click vào nút DOWNLOAD ở trên

Tài liệu đính kèm:

  • doc4_de_kiem_tra_hoc_ky_i_mon_tieng_anh_lop_12.doc

Nội dung text: 4 Đề kiểm tra học kỳ I môn Tiếng Anh Lớp 12

  1. TEST 01-07 Mark the letter A, B, C, or D on your answer sheet to indicate the word whose underlined part differs from the other three in pronunciation in each of the following questions. Question 1. A. part B. superstar C. harvest D. particular Question 2. A. reliable B. liquid C. vital D. suicidal Mark the letter A, B, C, or D on your answer sheet to indicate the word that differs from the other three in the position of primary stress in each of the following questions. Question 3. A. future B. involve C. prospect D. guidance Question 4. A. laptop B. mobile C. engine D. device Mark the letter A, B, C, or D on your answer sheet to indicate the underlined part that needs correction in the each of following questions. Question 5. A huge amount (A) of tourists (B) come (C) to this city (D) every year. Question 6. Of (A) all seashore plants, seaweeds are best (B) able to tolerate long periods out of water, followed by (C) long periods covering (D) by water. Question 7. Lack of (A) animal protein in (B) the human diet is a serious (C) cause of the (D) malnutrition. Mark the letter A, B, C, or D on your answer sheet to indicate the correct answer to each of the following questions. Question 8. A recent survey has shown that supporters of equal partnership in marriage are in the ___. A. crowd B. particular C. obligation D. majority Question 9. John asked me ___ in English. A. what that word means B. what did this word mean C. what does this word mean D. what that word meant Question 10. “I have gone to the doctor's to have a check up.” – “You ___.You just had your check-up last week!” A. didn't need to go B. needn’t have gone C. needn’t go D. don’t need to go Question 11. Body language is a potent form of ___ communication. A. verbal B. oral C. non-verbal D. tongue Question 12. ___ we work with her, we get confused because of her fast speaking pace. A. So that B. Although C. Whenever D. Lest Question 13. ___ umbrella should not be used during ___ thunderstorm. A. A/ a B. The/a C. An/a D. No article/ a Question 14. Many people are trying their best to study and work in the hope that they will ___ fame and fortune in the near future. A. lose B. achieve C. collect D. remove Question 15. John proposed ___ Mary on a fine day at the crowded beach in their hometown. A. that they married B. getting married C. to have married D. to marry Question 16. The students in class were made ___ very hard. A. learning B. learn C. to learn D. having learnt Question 17. I don’t think T-shirts and jeans will ever go ___ of style among young people. A. away B. out C. off D. down Question 18. Her mother’s dream ___ a family doctor will be fulfilled in six years. A. by B. with C. of D. at Question 19. They ___ their service up to now. A. didn’t do B. haven’t done C. don’t do D. aren’t doing 1
  2. Mark the letter A, B, C, or D on your answer sheet to indicate the most suitable response to complete each of the following exchanges. Question 20. Mary is talking to a porter in the hotel lobby. Porter: “Shall I help you with your suitcase?” - Mary: “___” A. Not a chance. B. That's very kind of you C. I can’t agree more. D. What a pity! Question 21. Hoa is asking Hai, who is sitting at a corner of the room, seeming too shy. Hoa: “Why aren’t you taking part in our activities? ___” - Hai: “Yes. I can. Certainly.” A. Could you please show me how to get to the nearest post office? B. Shall I take your hat off? C. Can I help you? D. Can you help me with these decorations? Mark the letter A, B, C, or D on your answer sheet to indicate the word(s) CLOSEST in meaning to the underlined word(s) in each of the following questions. Question 22. My wife was so keen on the picture that she paid through her nose for it. A. paid nothing B. turned a deaf ear C. was offered D. paid much more than usual Question 23. In order to better understand English, the students incorporated grammar, vocabulary, and speaking in their studies. A. combined B. granted C. scrutinized D. skipped Mark the letter A, B, C, or D on your answer sheet to indicate the word(s) OPPOSITE in meaning to the underlined word(s) in each of the following questions. Question 24. People in Vietnam nod their head to show that they agree with something. A. blink B. shake C. wave D. slump Question 25. The distinction between schooling and education implied by this remark is very important. A. odd B. implicit C. obscure D. explicit Mark the letter A, B, C, or D on your answer sheet to indicate the sentence that is closest in meaning to each of the following questions. Question 26. It is a pity that I can’t speak English as a native speaker. A. I wish I could speak English as a native speaker. B. I wish I couldn’t speak English as a native speaker. C. I wish I can speak English as a native speaker. D. I wish I have spoken English as a native speaker. Question 27. “What language do you find the most difficult to learn of all?” Nhung asked Ha. A. Nhung asked Ha what language Ha found the most difficult to learn of all. B. Nhung wanted to know what language they founded the most difficult to learn of all. C. Nhung asked Ha what language did you find the most difficult to learn of all. D. Nhung asked Ha what language you found the most difficult to learn of all. Question 28. The music was so loud that we had to shout to each other. A. We didn’t say loud enough for us to hear each other. B. The music was too loud that we had to shout to each other. C. We didn’t say loud enough to hear each other. D. Because of loud music, we had to shout to each other. Mark the letter A, B, C, or D on your answer sheet to indicate the sentence that best combines each pair of sentences in the following questions. 2
  3. Question 29. She tried very hard to pass the driving test. She could hardly pass it. A. Although she didn’t try hard to pass the driving test, she could pass it. B. Despite being able to pass the driving test, she didn’t pass it. C. Hard as she tried, she could hardly hard pass the driving test. D. She tried very hard, so she passed the driving test satisfactorily. Question 30. We didn’t want to spend a lot of money. We stayed in a cheap hotel. A. In stead of spending a lot of money, we stayed in a cheap hotel. B. In spite of spending a lot of money, we stayed in a cheap hotel. C. We stayed in a cheap hotel, but we had to spend a lot of money. D. We didn’t want to spend a lot of money. We stayed in the cheap hotel. Read the following passage and mark the letter A, B, C, or D on your answer sheet to indicate the correct word or phrase that best fits each of the numbered blanks from 31 to 35. In this age of (31) ___ telephone networks and electronic mail, it seems that fewer and fewer people are taking time to sit down and write letters to friends and relatives. For hundreds of years, letters were the only way to keep (32) ___ people who were any distance away and letter-writing was seen as an important skill for all learned people to master. Gradually, (33) ___, the importance of writing letters is decreasing to a point that majority of us have to make a special effort to turn out something worthwhile when we apply for a job or make a complaint. In business circles the tendency is for routine communications to become shorter. Even though clients may appreciate a detailed letter, an employee who sends out long letters is often regarded as (34) ___. Many people prefer the telephone in all circumstances and its speed is essential in many situations, but (35) ___ have you put the telephone down, dissatisfied with what you have managed to say? I don’t think I’ll throw my pen away yet. Question 31. A. advanced B. progressive C. highly-developed D. all are correct Question 32. A. on good terms with B. in step with C. in contact with D. in favour of Question 33. A. for example B. therefore C. however D. in short Question 34. A. impossible B. unusual C. inefficient D. unimportant Question 35. A. how about B. how often C. how much D. how long Read the following passage and mark the letter A, B, C, or D on your answer sheet to indicate the correct answer to each of the questions from 36 to 42. Arid regions in the southwestern United States have become increasingly inviting playgrounds for the growing number of recreation seekers who own vehicles such as motorcycles or powered trail bikes and indulge in hill-climbing contests or in carving new trails in the desert. But recent scientific studies show that these off-road vehicles can cause damage to desert landscapes that has long-range effects on the area’s water- conserving characteristics and on the entire ecology, both plant and animal. Research by scientists in the western Mojave Desert in California revealed that the compaction of the sandy arid soil resulting from the passage of just one motorcycle markedly reduced the infiltration ability of the soil and created a stream of rain runoff water that eroded the hillside surface. In addition, the researchers discovered that the soil compaction caused by the off-road vehicles often killed native plant species and resulted in the invasion of different plant species within a few years. The native perennial species required many more years before they showed signs of returning. The scientists calculated that roughly a century would be required for the infiltration capacity of the Mojave soil to be restored after being compacted by vehicles. Question 36. What is the main topic of the passage? A. Problems caused by recreational vehicles B. Types of off-road vehicles C. Plants of the southwestern desert D. The increasing number of recreation seekers Question 37. According to the passage, what is being damaged? A. Motorcycles B. The desert landscape C. Roads through the desert D. New plant species 3
  4. Question 38. According to the passage, the damage to plants is ___. A. unnoticeable B. superficial C. long-lasting D. irreparable Question 39. According to the passage, what happens when the soil is compacted? A. Little water seeps through B. Better roads are made C. Water is conserved D. Deserts are expanded Question 40. What is happening to the desert hillsides? A. The topsoil is being eroded C. There are fewer types of plants growing on them B. The surface is being irrigated D. There are fewer streams running through them Question 41. According to the passage, what is happening to native plants in these areas? A. They are becoming more compact B. They are adapting C. They are invading other areas D. They are dying Question 42. It can be inferred that which of the following people would probably be most alarmed by the scientists’ findings? A. Historians B. Mapmakers C. Farmer D. Ecologists Read the following passage and mark the letter A, B, C, or D on your answer sheet to indicate the correct answer to each of the questions from 43 to 50. Another critical factor that plays a part in susceptibility to colds is age. A study done by the University of Michigan School of Public Health revealed particulars that seem to hold true for the general population. Infants are the most cold-ridden group, averaging more than six colds in their first year. Boys have more colds than girls up to age three. After the age of three, girls are more susceptible than boy's , and teenage girls average three colds a year to boy’s two. The general incidence of continues to decline into maturity. Elderly people who are in good health have as few as one or two colds annually. One exception is founds among people in the twenties, especially women, who show a rise in cold infections, because people in this age group are most likely to have young children. Adults who delay having children until thirties forties experience the same sudden increase in cold infections. The study also found that economics play an important role. As income increases, the frequency at which are reported in the family decreases. Families with the lowest income suffer about a third more colds than families at the lower end. Lower income generally forces people to live in more cramped quarters than those typically occupied by wealthier by wealthier people, and crowding increases the opportunities for the cold virus to travel from person to person. Low income may also adversely influence diet. The degree to which poor nutrition affects susceptibility to colds is not yet clearly established, but an inadequate diet is suspected of lowering resistance. Question 43: Which of the following is closet in meaning to the word “particulars” in line 2? A. Minor errors B. specific facts C. small distinctions D. individual people Question 44: What does the author claim about the study discussed in the passage? A. It contains many inconsistencies B. It specializes in children C. It contradicts the results of earlier studies in the field D. Its results apparently are relevant for the population as a whole Question 45: It maybe inferred from the passage that which of the following groups of people is most likely to catch colds? A. infant boys B. young girls C. teenage boys D. elderly women Question 46: There is information in the second paragraph of the passage to support which of the following conclusions? A. Men are more susceptible to cold than women B. Children infect their parents with colds. C. People who live in a cold climate have more colds than those who live in a warm one. 4
  5. D. People who don’t have children are more susceptible to colds than those who do. Question 47: The phrase “in this age group” refers to ___. A. Infants B. People in their twenties C. People in their thirties and forties D. Elderly people Question 48: The author’s main purpose in writing the last paragraph of the passage is to ___. A. Explain how cold viruses are transmitted B. Prove that a poor diet cause colds C. Discuss the relationship between income and frequency of colds D. Discuss the distribution of income among the people in the study Question 49: The word “cramped” is closest in meaning to ___. A. cheap B. crowded C. depressing D. simple Question 50: The author’s tone in this passage could best be described as ___. A. Neutral and objective B. Humorous C. Tentative but interested D. Highly critical TEST 02-08 Mark the letter A, B, C, or D on to indicate the correct answer to each of the following questions. Question 1:___ every major judo title, Mark retired from international competition. A. When he won B. Having wonC. WinningD. On winning Question 2: The opposition will be elected into government at the next election, without a___ of a doubt A. shade B. shadowC. benefitD. hue Question 3: her fiction describes women in unhappy marriages. A. Many of B. A large number ofC. A great volume ofD. Much of Question 4: It seems that the world record for this event is almost impossible to . A. getB. beatC. take D. achieve Question 5: The smell was so bad that it completely us off our food. A. set B. tookC. gotD. put Question 6: He has been waiting for this letter for days, and at it has come. A. lastB. the end C. present D. the moment Question 7: It is opportunity to see African wildlife in its natural environment. A. an unique B. a uniqueC. the uniqueD. unique Question 8: " Why don’t we ask Martin to chair the meeting?” "Well, suppose to ask him, do you think he would accept?” A. were weB. we haveC. we tryD. we were Question 9: Delegates will meet with from industry and the government. A. RepresentedB. representativeC. representativesD. representers Question 10: Determining the mineral content of soil samples is an exacting process; ___ experts must perform detail tests to analyze soil specimens. A. so thatB. HoweverC. afterwardsD. therefore Question 11: Peter: “Is it important?” Thomas: “___” A. Not on your life! B. It’s a matter of life and death! C. No worry, that’s nothing. D. It’s ridiculous. Question 12: Ann. “Does the global warming worry you? Mathew. “___”. A. What a shame! B. Oh, it’s hotter and hotter C. I can’t bear to think about it.D. I don’t like hot weather, in the garden all day. 5
  6. Question 13: You look exhausted. You ___ in the garden all day. A. must have worked B. must be working C. can’t have workedD. must have been working Question 14: I’ve never really enjoyed going to the ballet or the opera; they’re not really my___. A. piece of cakeB. sweets and candy C. biscuit D. cup of tea Mark the letter A, B, C, or D on your answer sheet to indicate the word whose underlined part that differs from the other three in the pronunciation in each of the following questions. Question 15: A. spear B. gear C. fearD. pear Question 16: A. swordB. sweetC. swearD. swing Mark the letter A, B, C, or D to show the underlined part that needs correction in each of the following questions. Question 17: The composer Verdi has written the opera Aiđa to celebrate the opening the Suez Canal, A B C but the opera was not performed until 1871. D Question 18: Wealthy people have always desired and wear precious stones because their beauty A B C is lasting. D Question 19: Every city in the United States has traffic problems because the amount of cars A B C on American streets and highways is increasing every year. D Mark the letter A, B, C, or D to indicate the word or phrase that is OPPOSITE in meaning to the underlined part in each of the following questions. Question 20: His physical condition was not an impediment to his career as a violinist A. hindranceB. obstructionC. furtheranceD. setback Question 21: The ship went down although strenuous efforts were made to save it. A. exhausting B. forcefulC. energeticD. half-hearted Read the following passage and mark the letter A, B, C, or D to indicate the correct word(s) for each of the following blanks. It can take along time to become successful in your chosen field, however talented you are One thing you have to be (22) ___ of is that you will face criticism along the way. The world is full of people who would rather say something negative than positive. If you’ve made up your mind to achieve a certain goal, such as writing a novel don’t let the negative criticism of others prevent you from reaching your purpose and let constructive criticism have a positive (23) ___ on your work. If someone says you’re totally lacking talent ignore them. That’s negative criticism. If, however, someone advises you to revise your work and gives you good reasons for doing so, you should (24) ___ their suggestions carefully. There are many film stars who were once out of (25) ___ .Therearemanyfamous novelists who made a complete mess of their first novel - or who didn’t, but had to keep on approaching hundreds of publishers before they could get it published. Being successful does depend on luck, to a certain extent. But things are more likely to (26) ___ well if you persevere an s ay 6
  7. Question 22: A. kept in mindB. cleverC. intelligent D. aware Question 23: A. affectB. effectC. result D. change Question 24: A. cautiousB. considerC. reckon D. remember Question 25: A. careerB. business C. job D. work Mark the letter A, B, C, or D to indicate the word that differs from the rest in the position of the main stress in each of the following questions. Question 27: A. mutually B. apologize C. permissionD. reaction Question 28: A. survival B. industry C. endangered D. commercial Mark the letter A, B, C, or D to indicate the word or phrase that is CLOSEST in meaning to the underlined pan in each of the following questions. Question 29: She is always diplomatic when she deals with angry students. A. strictB. outspokenC. tactfulD. firm Question 30: Nobel's original legacy of nine million dollars was invested, and the interest on this sum is used for the awards which vary from $30, 000 to $125, 000. A. fortuneB. prizeC. heritageD. bequest Read the following passage and mark the letter A, B, C or D to indicate the correct answer to each of the following questions. Of all modern instruments, the violin is apparently one of the simplest. It consist in essence of a hollow, varnished wooden sound box, or resonator, and a long neck covered with a fingerboard, along which four strings are stretched at high tension. The beauty of design, shape, and decoration is no accident, the proportions of the instrument are determined entirely by acoustical considerations. Its simplicity of appearance is deceptive. About 70 parts are involved in the construction of a violin. Its tone and its outstanding range of expressiveness make it an ideal solo instrument. No less important, however, is its role as an orchestral and chamber instrument In I combination with the larger and deeper-sounding members of the same family, the violins form the nucleus of the modem symphony orchestra. The violin has been in existence since about 1550. Its importance as an instrument in its own right dates from the early 1600’s, when it first became standard in Italian opera orchestras. Its stature as an orchestral instrument was raised further when in 1626 Louis XIII of France established at his court the orchestra known as Les vinq-quatre violons du Roy (The King's 24 Violins), which was to become widely famous later in the century. In its early history, the violin had a dull and rather quiet tone resulting from the fact that the strings were thick and were attached to the body of the instrument very loosely. During the eighteenth and nineteenth century exciting technical changes were inspired by such composer-violinists as Vivaldi and Tartini. Theừ instrumental compositions demanded a fuller, clearer, and more brilliant tone that was produced by using thinner strings and a far higher string tension. Small changes had to be made to the violin's internal structure and to the fingerboard so that they could withstand the extra strain. Accordingly, a higher standard of performance was achieved, in terms of both facility and interpretation. Left-hand technique was considerably elaborated, and new fingering patterns on the fingerboard were developed for very high notes. Question 31: The word "standard" is closest in meaning to . A. practicalB. customary C. possibleD. unusual Question 32: "The King’s 24 Violins" is mentioned to illustrate. A. the competition in the 1600's between French and Italian orchestras B. how the violin became a renowned instrument C. the superiority of French violins D. why the violin was considered the only instrument suitable to be played by royalty 7
  8. Question 33: What is the main idea presented in paragraph 3? A. The violin is probably the best known and most widely distributed musical instrument in the world. B. The violin has been modified to fit its evolving musical functions. C. The violin had reached the height of its popularity by the middle of the eighteenth century. D. The technique of playing the violin has remained essentially the same since the 1600's. Question 34: The word "they" in paragraph 3 refers to . A. Civaldi and TartiniB. internal structure and fingerboard C. thinner strings and a higher string tensionD. small changes Question 35: According to the passage, early violins were different from modem violins in that early violins A. were heavierB. produced softer tones C. were easier to playD. broke down more easily Question 36: According to the passage, which of the following contributes to a dull sound being produced by a violin? A. A long fingerboard B. A small bodyC. High string tension D. Thick strings Question 37: All of the following are mentioned in the passage as contributing to the ability to play modern violin music EXCEPT . A. use of rare wood for the fingerboard and neck B. different ways to use the fingers to play very high notes C. more complicated techniques for the left hand D. minor alterations to the structure of the instrument Read the following passage and mark the letter A, B, C, or D to indicate the correct word(s) for each of the following blanks. Pollutants are clogging up the atmosphere above our planet and trapping the sun's heat. That makes our planet warmer, which causes glaciers to melt, crazy weather patterns to develop, and natural disasters spread like wildfires. But there are other consequences that people don't always associate with climate change. One surprising fact about global warming is that it makes people more aggressive. As the world gets hotter, tempers also get fierier. A new study found that violent crimes and even war become more likely as temperatures rise. Heart rates tend to be elevated in warm weather, so people are prepped for a physical response to a situation. That’s not to say we're headed for a violent end as Earth warms up, though. Advances in everything from technology to health have sparked a decline in conflict. It's just that global warming might be slowing that decline. Question 38: What is trapping the sun’s heat? A. The atmosphereB. PollutantC. GlaciersD. The atmosphere above our planet Question 39: According to the paragraph 1, all of the following are results of our planet getting warmer EXCEPT A. Melting glaciersB. Increasing greenhouse gas C. Natural disasters like wildfiresD. Crazy weather patterns to develop Question 40: One surprising fact about global warming surprising fact about global warming is that A. it could sink citiesB. it makes people more aggressive C. it makes mountains tallerD. It will last for centuries Question 41: What does the new study found about the causes of rising temperatures? A. People becomes more healthyB. Plants bloom earlier in the winter C. Violent crimes are likely to happenD. Sea level rises Question 42: Why are people prepped for a physical response to a situation? A. Because heart rates tend to be elevated in warm weather 8
  9. B. Because there are a lot of surprising facts C. Because their hearts are weaker due to the rising temperatures D. Because they are more aggressive now Question 43: The author uses the phrase “headed for” to mean A. To changeB. To experience something soon C. To go forD. To be good at Question 44: The word “elevated” is closest in meaning to A. dropped B. grew C. enlargedD. raised Question 45: What is the topic of the passage? A. A global warming surprising factB. Aggressive modem people C. The bad effects of global warmingD. A study on global warming Mark the letter A, B, C, or D to indicate the sentence (hat best combines each pair of sentences in the following questions. Question 46: We cut down many forests. The Earth becomes hot. A. The more forests we cut down, the hotter the Earth becomes. B. The more we cut down forests, the hotter the Earth becomes, C. The more forests we cut down, the Earth becomes hotter. D. The more we cut down forests, the Earth becomes hotter. Question 47: The new restaurant looks good. However, it seems to have few costumers. A. In spite of its appearance, the new restaurant does not appear to attract much business. B. The new restaurant would have more customers if it looked better. C. In order to get more business, the new restaurant should improve its appearance. D. If it had a few more customers, the new restaurant would look better. Mark the letter A, B, C, or D on your answer sheet to choose the sentence which is closest in meaning to the given one. Question 48: Please arrive early because we want to be able to start our meeting early A. Please arrive early so that we can start our meeting early. B. Please arrive early so that we are able to start our meeting early C. Please arrive early so that we would be able to start our meeting early D. Please arrive early for us to be able to start our meeting early Question 49: There was no need for you to have left the house in such weather. A. You needn’t leave the house in such weather. B. You needn’t have left the house in such weather. C. You didn’t have to leave the house in such weather. D. You didn’t need to leave the house in such weather. Question 50: The concert went like a bomb. A. The concert was a disasterB. There was a bomb in the concert C. The concert was very violentD. The concert was extremely successful. TEST 03-09 Mark the letter A, B, C, or D on your answer sheet to indicate the word whose underlined part differs from the other three in pronunciation in each of the following questions. Question 1: A. insects B. fields C. frogs D. villagers 9
  10. Question 2: A. village B. luggage C. page D. damage Mark the letter A, B, C, or D on your answer sheet to indicate the word that differs from the other three in the position of primary stress in each of the following questions. Question 3: A. surgeon B. surprise C. surplus D. surface Question 4: A. apology B. geography C. experience D. preparation Mark the letter A, B, C, or D on your answer sheet to indicate the underlined part that needs correction in each of the following questions. Question 5: Jack works as a lawyer and he earns twice as much than his brother. A B C D Question 6: The change from day to night results in the rotation of the Earth. A B C D Question 7: Because the student spent too many time surfing the Internet, he didn't study well. A B C D Mark the letter A, B, C, or D on your answer sheet to indicate the correct answer to each of the following questions. Question 8: Don't try to phone me! By the time you read this letter, ___. A. I have left B. I will have left C. I will leave D. I'm leaving Question 9: Nobody says a word about the accident, ___? A. does he B. doesn't he C. do they D. don't they Question 10: Stress, ___is a psychological problem, may lead to physical illness. A. which B. what C. that D. whose Question 11: She gave me a ___ box. A. jewelry metal small square B. jewelry small metal square C. small square jewelry metal D. small square metal jewelry Question 12: ___ his good work and manners, he didn't get a promotion. A. Because of B. In spite of C. Even though D. As a result of Question 13: You ___ your room carelessly because it is still very untidy. A. shouldn't clean B. could clean C. must have cleaned D. can't have cleaned Question 14: She lost her job because she was ___ . She made far too many mistakes. A. rash B. incautious C. inefficient D. impulsive Question 15: My father put ___ the money to buy a new house. A. in B. up C. on D. out Question 16: I like his essay because it's very___. A. imagination B. imaginable C. imaginative D. imaginary Question 17: We sailed slowly down___ Red river. A. a B. an C. the D. 0 Question 18: Only when you become a parent___ what true responsibility is. A. you will understand B. will you understand C. you understand D. don't you understand Question 19: Prices are rising quickly everywhere. The ___ seem to go up every day. A. standard of living B. quality of life C. annual income D. cost of living Question 20: Julie and Nina are playing in the garden. Julie: "Look at this beautiful butterfly!" Nina: "___” 10
  11. A. Where? I don't see it. B. Yes, please. C. Don't worry. D. No, it's your turn. Question 21: Sarah's giving John a gift on his birthday. John: "Thank you very much for the lovely gift." Sarah:”___" A. You're welcome. B. Congratulations. C. I'm not interested. D. No, I don't think so Mark the letter A, B, C, or D on your answer sheet to indicate the word(s) CLOSEST in meaning to the underlined word(s) in each of the following questions. Question 22: The student was unable to provide an acceptable explanation. A. simple B. alternative C. additional D. satisfactory Question 23: In Roman times, a sophisticated technology brought running water into private homes and public bathhouses. A. experienced B. complicated C. worldly D. naive Mark the letter A, 8, C, or D on your answer sheet to indicate the word(s) OPPOSITE in meaning to the underlined word(s) in each of the following questions. Question 24: Our traditions are very ancient and our people are very proud of them A. old B. young C. modern D. real Question 25: The milk is delivered at 6 A.M so we always have fresh milk for breakfast. A. sour B. old C. dirty D. new Mark the letter A, B, C, or D on your answer sheet to indicate the sentence that is closest in meaning to each of the following questions. Question 26: Friendly though he may seem, he's not to be trusted. A. However he seems friendly, he's not to be trusted. B. However friendly he seems, he's not to be trusted. C. He is too friendly to be trusted. D. He may have friends, but he is not to be trusted. Question 27: "Leave my house or I'll call the police!" shouted the lady to the man. A. The lady informed the man that she would call the police if he didn't leave her house. B. The lady said that she would call the police if the man didn't leave her house. C. The lady told the man that she would call the police if he didn't leave her house. D. The lady threatened to call the police if the man didn't leave her house. Question 28: Their chances of success are very small. A. It's not very likely that they will succeed. B. There is possibility that they will achieve success. C. They will certainly be very successful. D. They have no chances, so they are not successful. Mark the letter A, B, C, or D on your answer sheet to indicate the sentence that best combines each pair of sentences in the following questions. Question 29: In Paris, Hemingway met many of his old friends. They encouraged him in his writing career. A. In Paris, Hemingway encouraged many of his old friends who met him in his writing career. B. In Paris, Hemingway encouraged his writing career to meet many of his old friends. C. In Paris, Hemingway met his writing career which many of his old friends encouraged. D. In Paris, Hemingway was encouraged by many of his old friends in his writing career. 11
  12. Question 30: Jim is my best friend. I borrowed his car yesterday. A. Jim, whose car I borrowed yesterday, is my best friend. B. Jim, who car I borrowed yesterday, is my best friend. C. Jim, who is my best friend, borrowed my car yesterday. D. Jim, whose car I lent yesterday, is my best friend Read the following passage and mark the letter A, B, C, or D on your answer sheet to indicate the correct word or phrase that best fits each of the numbered blanks from 31 to 35. Children (31)___ appear intelligent and have normal sight and hearing may nevertheless have learning disability such as dyslexia, difficulty in reading; dysgraphia, difficulty in writing; dyscalculia, difficulty with numbers; and auditory-memory problem that (32)___the child from remembering what has just been said. Considered an "invisible" (33)___, such learning disabilities can be detected by alert parents before the children go to school. (34)___ the child at about thirty months is not developing normal language skills, something is amiss. A child who cannot do puzzles or put pegs in holes lacks perceptual- motor skills. Kindergarteners should (35)___the ABCs. First-graders may commonly reverse their letters, writing a d or a b, but if they are still doing this at the start of second grade, they should be tested for learning disabilities. Proper and early treatment is essential. Question 31: A. whom B. who C. whose D. they Question 32: A. avoid B. help C. encourage D. prevent Question 33: A. barrier B. retard C. disabled D. handicap Question 34: A. If B. When C. Although D. Because Question 35: A. read B. remember C. recognize D. pronounce Read the following passage and mark the letter A, B, C, or D on your answer sheet to indicate the correct answer to each of the questions from 36 to 42. The technology of the North American colonies did not differ strikingly from that of Europe, but in one respect, the colonists enjoyed a great advantage. Especially by comparison with Britain, Americans had a wonderfully plentiful supply of wood. The first colonists did not, as many people imagine, find an entire continent covered by a climax forest. Even along the Atlantic seaboard, the forest was broken at many points. Nevertheless, all sorts of fine trees abounded, and through the early colonial period, those who pushed westward encountered new forests. By the end of the colonial era, the price of wood had risen slightly in eastern cities, but wood was still extremely abundant. The availability of wood brought advantages that have seldom been appreciated. Wood was a foundation of the economy. Houses and all manner of buildings were made of wood to a degree unknown in Britain. Secondly, wood was used as fuel for heating and cooking. Thirdly, it was used as the source of important industrial compounds, such as potash, an industrial alkali; charcoal, a component of gunpowder; and tannic acid, used for tanning leather. The supply of wood conferred advantages but had some negative aspects as well. Iron at that time was produced by heating iron ore with charcoal. Because Britain was so stripped of trees, she was unable to exploit her rich iron mines. But the American colonies had both iron ore and wood; iron production was encouraged and became successful. However, when Britain developed coke smelting, the Colonies did not follow suit because they had plenty of wood and besides, charcoal iron was stronger than coke iron. Coke smelting led to technologic innovations and was linked to the emergence of the Industrial Revolution. In the early nineteenth century, the former colonies lagged behind Britain in industrial development because their supply of wood led them to cling to charcoal iron. Question 36: What does the passage mainly discuss? A. The advantages of using wood in the colonies. B. The effects of an abundance of wood on the colonies. 12
  13. C. The roots of the Industrial Revolution. D. The difference between charcoal iron and coke iron. Question 37: The word "strikingly" in the first paragraph is closest in meaning to___. A. realistically B. dramatically C. completely D. immediately Question 38: Which of the following is a common assumption about the forests of North America during the colonial period? A. They contained only a few types of trees. B. They existed only along the Atlantic seaboard. C. They had little or no economic value. D. They covered the entire continent. Question 39: According to the passage, by the end of the colonial period, the price of wood in eastern cities ___. A. rose quickly because wood was becoming so scarce B. was much higher than it was in Britain C. was slightly higher than in previous years D. decreased rapidly because of lower demand for wood Question 40: According to the passage, why was the use of coke smelting advantageous? A. It led to advances in technology. C. It produced a stronger type of iron than wood smelting. B. It was less expensive than wood smelting. D. It stimulated the demand for wood Question 41: Why does the author mention "gunpowder" in paragraph 3? A. To illustrate the negative aspects of some industrial processes B. To give an example of a product made with wood compounds C. To remind readers that the colonial era ended in warfare. D. To suggest that wood was not the only important product of the colonies. Question 42: The phrase "follow suit" in paragraph 4 means . A. do the same thing B. make an attempt C. have the opportunity D. take a risk Read the following passage and mark the letter A, B, C, or D on your answer sheet to indicate the correct answer to each of the questions from 43 to 50. There are desert plants which survive the dry season in the form of inactive seeds. There are also desert insects which survive as inactive larvae. In addition, difficult as it is to believe, there are desert fish which can survive through years of drought in the form of inactive eggs. These are the shrimps that live in the Mojave Desert, an intensely dry region in the south-west of the United States where shade temperatures of over 50C are often recorded. The eggs of the Mojave shrimps are the size and have the appearance of grains of sand. When sufficient spring rain falls to form a lake, once every two to five years, these eggs hatch. Then the water is soon filled with millions of tiny shrimps about a millimetre long which feed on tiny plant and animal organisms which also grow in the temporary desert lake. Within a week, the shrimps grow from their original 1 millimeter to a length of about 1.5 centimetres. Throughout the time that the shrimps are rapidly maturing, the water in the lake equally rapidly evaporates. Therefore, for the shrimps it is a race against time. By the twelfth day, however, when they are about 3 centimetre long, hundreds of tiny eggs form on the underbodies of the females. Usually by this time, all that remains of the lake is a large, muddy patch of wet soil. On the thirteenth day and the next, during the final hours of their brief lives, the shrimps lay their eggs in the mud. Then, having ensured that their species will survive, the shrimps die as the last of the water evaporates. If sufficient rain falls the next year to form another lake, the eggs hatch, and once again the shrimps pass rapidly through their cycle of growth, adulthood, egg-laying, and death. Some years there is insufficient rain to form a lake: in this case, the eggs will remain dormant for another year, or even longer if necessary. 13
  14. Very, very occasionally, perhaps twice in a hundred years, sufficient rain falls to form a deep lake that lasts a month or more. In this case, the species passes through two cycles of growth, egg-laying, and death. Thus, on such occasions, the species multiplies considerably, which further ensures its survival. Question 43: Which is the best title for this passage? A. Life in the Mojave Desert B. Life Cycle of the Mojave Shrimps C. The Survival of Desert Insects D. Water and Life in the Desert Question 44: The word "these" in the first paragraph refers to___. A. plans B. eggs C. insects D. fish Question 45: According to the passage, the eggs originate___. A. in the sand B. on the female C. in the mud D. in the lakes Question 46: By saying "for the shrimps it is a race against time" the author means___. A. they have to swim fast to avoid danger in the rapidly evaporating lake B. they have to swim fast to catch the animal organisms on which they survive C. they have to multiply as many as possible within thirteen days D. they have to complete their life cycle within a short span of time permitted by the environment Question 47: The word "dormant" in the fourth paragraph most probably means___. A. inactive B. strong C. dead D. empty Question 48: According to the passage, approximately how long does a shrimp live? A. about a week B. about 13 days C. about a month D. not given Question 49: What is unusual about the Mojave Shrimps? A. Their eggs can survive in an intensively dry region B. Their lives are very brief. C. They feed on plant and animal organisms. D. They lay their eggs in the mud only. Question 50: It may be inferred from the passage that ___, A. appearance and size are most important for life to survive in the desert B. a species must be able to multiply quickly in order to survive C. for some species one life cycle in a year is enough to survive the desert drought D. some species develop a unique life pattern to survive in extremely harsh conditions TEST 04-10 Mark the letter A, B, C, or D on your answer sheet to indicate the word whose underlined part differs from the other three in pronunciation in each of the following questions. Question 1. A. valentineB. imagineC. disciplineD. determine Question 2. A. linkedB. declared C. finished D. developed Mark the letter A, B, C, or D on your answer sheet to indicate the word that differs from the other three in the position of primary stress in each of the following questions. Question 3. A. complimentB. convenientC. tsunamiD. tradition Question 4. A. wildlifeB. injureC. degreeD. passage Mark the letter A, B, C, or D on your answer sheet to indicate the word(s) CLOSEST in meaning to the underlined word(s) in each of the following questions. Question 5. The year-end party was out of this world. We had never tasted such delicious food. A. enormous B. terrific C. strange D. awful Question 6. In some Western cultures, people who avoid eye contact in a conversation are regarded as untrustworthy. A. embarrassed B. unreasonable C. dependable D. unreliable 14
  15. Mark the letter A, B, C, or D on your answer sheet to indicate the word(s) OPPOSITE in meaning to the underlined word(s) in each of the following questions. Question 7. The dog is starving. It hasn’t got anything to eat for nearly a week. A. quite happy B. very hungry C. rather full D. extremely thirsty Question 8. Safety regulations have made it obligatory for all employees to wear protective clothing. A. popular B. optional C. common D. compulsory Mark the letter A, B, C, or D on your answer sheet to indicate the correct answer to each of the following questions. Question 9. The woman ___ gave me the book is my aunt. A. where B. when C. who D. which Question 10. After nine months without any rain, the country was facing one of the worst in the last fifty years. A. draughts B. floods C. eruptions D. droughts Question 11. Jack’s father bought him a(n) ___ bike as a birthday gift. A. brown Vietnamese expensive B. expensive Vietnamese brown C. Vietnamese expensive brown D. expensive brown Vietnamese Question 12. The harder they study, ___. A. the better results they get B. their results get better C. the better do their results get D. the best results they gets Question 13. We ___ full use of the fine weather and had a day out. A. took B. created C. did D. made Question 14. My neighbor, Nga, ___ to America last month. A. was moving B. moves C. moved D. has moved Question 15. In this game, you are not allowed to speak; You have to use non-verbal___. A. communicated B. communication C. communicate D. communicative Question 16. Telephone sometimes causes ___ between members of the family. A. arguments B. differences C. quarrel D. misunderstand Question 17. I have great ___for the young teachers who devote their life to teaching children in remote and mountainous area. A. intention B. admiration C. consideration D. sympathy Question 18. If I were you, I .ask the doctor for advice. A. can B. will C. must D. would Question 19. Have you read ___ novel we bought in the supermarket last Saturday? A. a B. the C. an D. no article Question 20. When you have anything important to do, do it straight away. Don’t put it___ and then feel regretful. A. down B. on C. off D. aside Mark the letter A, B, C, or D on your answer sheet to indicate the most suitable response to complete each of the following exchanges. Question 21. Ken and Laura are saying goodbye to each other after going to LOTTE Center. And they are going to have a date with each other later. - Laura: “Well, it’s getting late. Maybe we could get together sometime.” - Ken: “ ___” 15
  16. A. Nice to see back you. B. Take it easy. C. Sounds good. I’ll give you a call. D. Yes, I’ve enjoyed it. Question 22. Two friends, John and Laura are talking about the public transportation John: “Do you think that we should use public transportation to protect our environment?” Laura: “___ ”. A. Yes, it's an absurd idea B. There's no doubt about it C. Of course not. You bet D. Well, that's very surprising Mark the letter A, B, C, or D on your answer sheet to indicate the sentence that best combines each pair of sentences in the following questions. Question 23. The room had been painted in dark colors. It needed some bright lights. A. Having been painted in dark colors, the room needed some bright lights. B. Although the room needed some bright lights, it had been painted in dark colors. C. Because the room needed some bright lights, it had been painted in dark colors. D. Having painted in dark colors, the room needed some bright lights. Question 24. Sports help maintain lower body weight. Sports also promote self confidence. A. Sports help maintain lower body weight, but they promote self confidence. B. Sports help maintain lower body weight, and they promote self confidence. C. Sports help maintain lower body weight, for they promote self confidence. D. Sports help maintain lower body weight, or they promote self confidence. Mark the letter A, B, C, or D on your answer sheet to indicate the sentence that is closest in meaning to each of the following questions. Question 25. I couldn’t have made it without your help. A. I couldn’t have made it if you helped me. B. I couldn’t have made it if you had helped me. C. I couldn’t have made it if you hadn’t helped me. D. I couldn’t have made it if you didn’t help me. Question 26. The last time I played football was in 1991 A. I haven’t played football in 1991 B. I haven’t played football since 1991 C. I didn’t play football in 1991D. I last played football since 1991 Question 27. "Don't forget to turn off the light before leaving." A. He asked me to leave and turn off the light. B. He reminded me to turn off the light before leaving. C. He promised to turn off the light before leaving. D. He warned me about turning off the light before leaving. Mark the letter A, B, C, or D on your answer sheet to indicate the underlined part that needs correction in each of the following questions. Question 28. : Neither his parents nor his teacher were satisfied with his result when he was at high school. A B C D Question 29. I always make the children to pick up their toys. A B C D Question 30. Gray whales migrate 5,000 miles from Arctic waters to bays in California A B where they give birth for their calves. C D 16
  17. Read the following passage and mark the letter A, B, C, or D on your answer sheet to indicate the correct word or phrase that best fits each of the numbered blanks from 31 to 35. PEER PRESSURE Sociologists have been carrying out research into the social pressures of being a teenager. Many adolescents are unhappy at school because they find it difficult to (31)___ friends. This stress can bring on illness or result in poor grades. They may also worry about their appearance and often feel (32)___ enormous pressure to dress, talk and behave the same as (33)___. This phenomenon is called peer pressure, and it is very common in today’s society. Advertising contributes a lot to the social pressures teenagers feel. Advertisers know how important it is to feel that you belong to a group when you are in your teens, so they try ti persuade teenagers that certain products will make them (34)___ with their classmates. Sadly, many teenagers act irresponsibly and even do dangerous things just to make others accept them. Peer pressure is often the reason for teenage smoking, drug (35)___ or dangerous driving. Teenagers need to learn to say “no” to social pressure and to find friends who they can talk things over with when they have a problem. Question 31: A. keep B. create C. remain D. make Question 32: A.in B. under C. over D. below Question 33: A. another B. the other C. others D. other Question 34: A. common B. popular C. open D. favorite Question 35: A. overdose B. abuse C. addicted D. overuse Read the following passage and mark the letter A, B, C, or D on your answer sheet to indicate the correct answer to each of the questions from 36 to 42. Young people have always liked having their own language. Each generation has its own slang words, and they change faster than most adults can keep up with. That’s the point of slang, according to experts who study language and communication, and according to the young people themselves. “You don’t want your parents or teachers understanding everything you say,” says one 15-year-old. “We need our own space, too.” But nowadays it’s not a matter of just a few words. There is a whole new language developing. It’s happening online, and it’s happening at lightning speed. “It” is the language of Instant Messaging, or IM, also used in text messaging. For young people, it’s a fun and creative way to converse. And the cool thing for them is, most adults can’t understand it. “Young people have always been secretive,” says one communication expert. “It used to be passing notes under the desk in class. It’s a way of keeping in touch and keeping adults in the dark.” There isn’t only one IM language, either. Young people in every country have developed their own lingo, mainly based on sounds. For example, the Spanish question word ¿que? Become K? in IM language. In French, A+ (a plus) isn’t a good grade, it’s the equivalent of L8R (later). In Japan, teenage girls have created an entire language called Gal Go. It is a combination of different types of Japanese characters, and even boys of their generation find it impossible to figure out. The good news is that the preoccupation with IM – and with secrecy – doesn’t last forever. By the time they enter college, most young people are using a simpler version of IM language, with just a few common abbreviations everyone knows. By that time, IM is just a practical way to keep in touch with friends and colleagues, not a way to keep secrets from the older generation. Question 36: According to the passage, each generation of young people ___. A: uses the same slang words their parents did B: has its own slang vocabulary C: wants the older generation to understand them D: uses less slang than the previous generation Question 37: The 15-year-old suggests that young people ___. A: want their own rooms B: don't like talking to teachers C: need privacy D: don't understand parents or teachers Question 38: According to the passage, what is true about IM language? A: It is developing quickly B: It is developing slowly 17
  18. C: It is being learned by adults D: It is easy to learn Question 39: It is stated in the passage that IM language ___. A: is the same all over the world B: is used more often by girls C: is simpler than Japanese D: differs depending on the country it's used in Question 40: The word "lingo" in the passage would probably mean ___. A: language B: alphabet C: message D: IM device Question 41: The word "It" in the passage refers to ___. A: Gal Go B: A+ C: L8R D: K? Question 42: What usually happened by the time young people enter college? A: They forget how to use IM B: They use a less complicated form of IM language C: They use a more complicated form of IM language D: They lose interest in keeping in touch Read the following passage and mark the letter A, B, C, or D on your answer sheet to indicate the correct answer to each of the questions from 43 to 50. A ghostly animal creeps silently through a Florida swamp. It’s a rare type of big cat known as a Florida panther, one of only 80 to 100 such panthers left in the world. Scientists must work to save these remaining panthers from extinction, but their secretive nature is making it difficult. They are attaching a special transmitter to each Florida panther so they can follow their movements, range, and habits. The technology these transmitters use is called satellite tracking. The scientists are dedicated professionals, but they are not pioneers. Their colleagues before them have attached tiny transmitters to many different kinds of wild animals, including birds, fish, and big cats. So the Florida scientists are using their methods. Firstly, they must trap and tranquillize the panther. Then, the transmitter is attached to a harness and strapped to the panther’s body. Each harness is custom designed and manually adjusted for panthers so it fits comfortably. Scientists don’t want the transmitter to interfere with the panther’s natural habits. The point of tracking them is to find out as much as they can about their natural habits. Once the tracking transmitter is in place, the information it records is sent to an orbiting satellite. The satellite can see the transmitter that is attached to the panther, even when it is out of sight of a biologist on the ground. The satellite collects information from the transmitter about the panther’s exact location. Once the information is received, it is transmitted back to Earth so it can be recorded by the scientists. Scientists can use the information in many different ways. They can follow the panthers on the ground and observe their behavior. Alternatively, they can use the information to determine the panthers’ ranges and to understand their social patterns. They can also track how environmental changes like weather conditions affect their movements and health. All this information can be used to provide ideal conditions for the endangered panthers. It can also be used to protect the panthers from danger. In the end, the panther silently slips away through the trees. Around its shoulders a transmitter is sending signals via satellite to a dedicated biologist. The biologist is using the information the transmitter provides to save the Florida panthers from extinction. With the help of satellite tracking these beautiful cats have a better chance of survival in the modern world. (Adapted from Question 43: What type of technology are scientists using to follow the panther? A: satellite tracking B: special transmitter C: trap and tranquilize D: tracking transmitter Question 44: According to the ready passage, what information does the satellite collect from the transmitter? A: information about the biologists on the ground B: information about the chance of panthers' survival 18
  19. C: information about the total number of panthers D: information about the panther's exact location Question 45: Which sentence about the tracking transmitter is true according to the passage? A: Each transmitter is different because it is specially designed for a specific purpose. B: The transmitter is inserted invisibly inside the panther's body C: Scientists can follow the panthers using the information from the transmitter. D: The information is not useful for learning about the panthers' habit or social patterns Question 46: Which sentence about the tracking transmitter is NOT true? A: Scientists use the information the transmitter provides to save the panthers. B: The Florida panther is the only animal to be attached with the transmitter C: The transmitter sends a signal to a satellite that can see the transmitter D: The transmitter is attached to a harness and strapped to the panther's body Question 47: The word "tranquilize" in the passage mostly mean ___. A: put it in a cage B: capture it quietly C: handle it gently D: make it calm and quiet Question 48: The phrase "custom designed" in the passage probably means ___. A: designed by famous designers B: specially made for panthers C: created by hand D: functioned according to a custom Question 49: How does the writer explain the use of the information from the tracking transmitter in paragraph 4? A: by suggesting the best way to use the information B: by listing the different ways the information can be used C: by discussing the value of the information D: by comparing the positive and negative points about the information Question 50: What can be the best title of the reading passage? A: Keeping Track B: Information Transmitter C: Animal Tranquilizer D: Understanding Wildlife TEST 05 –11 đề thi thử 2017 Mark theletter A, B,C, orD onyour answersheet to indicate the words whose pronunciation is different from the others in each of the following questions. Question 1: A. required B. blamed C. dissolved D. talked Question 2: A. attract B. argument C. apply D. vacancy Mark the letter A, B, C, or D on your answer sheet to indicate the word that differs from the other three in the position of the primary stress in each of the following questions. Question 3: A. approximate B. congratulate C. psychological D. biography Question 4: A. rhinoceros B. advertisement C. eventually D. dedicated Mark the letter A, B, C, or D on your answer sheet to indicate the underlined part that needs correction in each of the following questions Question 5: An increasing number of companies has changed dress codes, allowing employees to wear casual clothing in the work place. A. increasing B. has changed C. to wear D. in the work place Question 6: Among the earliest telescopes were Galilean telescopes, modeled after the simple instruments built by Galileo, the first person having used telescopes to study the stars and planets. A. telescopes B. were C. modeled D. having used 19
  20. Question 7: Mr. Lam insisted that we show our best knowledge, our enthusiasm, our keen on work and our sense of responsibility when working in a team. A. show B. keen on C. sense of D. when working Mark the letter A, B, C, or D on your answer sheet to indicate the correct answer to each of the following questions Question 8: In England, English, Math and Science are ___ subjects at school. A. more B. store C. score D. core Question 9: It’s ___ to transfer drugs in our country. A. legally B. illegally C. illegal D. legal Question 10: The guidelines in this book can help you become a ___ speaker. A. confide B. confident C. confidence D. self-confidence Question 11: If people drove more carefully, there ___ so many accidents on the road. A. would not be B. will not be C. wouldn’t have been D. aren’t Question 12: ___ he got top marks at high school, he never went to university. A. Despite B. Although C. Meanwhile D. Nevertheless Question 13: When Jack ___ me, I ___ a letter. A. phoned - has been writing B. has phoned - was writing C. phoned- was writing D. was phoning - wrote Question 14: ___ migrate long distances is well documented. A. That birds B. Birds C. A bird D. The birds Question 15: - Joe: “This file is very important. You should include it in our document.” - Paul: “I know. It is ___.” A. significant B. optional C. indistinct D. indispensable Question 16: This carpet really needs___. Can you do it for me, son? A. being cleaned B. clean C. cleaned D. cleaning Question 17: I realized ___ that he was a thief. A. eventually B. sooner or later C. at the beginning D. all along Question 18: Designers are experimenting with a new material ___ flexibly with lightness. A. combines B. combination of C. combining D. is combining Question 19: When she died, she gave ___ all her money to a charity for cats. A. on B. off C. out D. away Mark the letter A, B, C, or D on your answer sheet to indicate the most suitable response to complete each of the following exchanges Question 20: “That’s a beautiful dress you are wearing!” - “___” A. Can I have it? B. It’s in the wash C. I’m glad you like it D. Oh, no, not really Question 21: “Would you like some chocolate?” - “___” A. I love it B. Yes, please! C. Here you go D. Here you are Mark the letter A, B, C, or D on your answer sheet to indicate the word(s) CLOSEST in meaning to the underlined word(s) in each of the following questions Question 22: The fourth-year sociology class was a homogeneous group of university students. A. unrelated B. uniform C. distinguishable D. dreary Question 23: I only have time to tell you the main idea of it, not the details. A. fist B. gist C. twist D. list 20
  21. Mark the letter A, B, C, or D on your answer sheet to indicate the word(s) OPPOSITE in meaning to the underlined word(s) in each of the following questions Question 24: My mother is a caring woman and always thoughtful of others. A. rude B. inconsiderate C. inconsiderable D. critical Question 25: There is no excuse for your discourtesy. Think twice before you are going to say anything. A. bravery B. impoliteness C. politeness D. boldness Mark the letter A, B, C, or D on your answer sheet to indicate the sentence that is closest in meaning to each of the following questions Question26: If there hadn’t been such a strong wind, it would not have been so difficult to put out the fire. A. It was the strong wind which made it difficult for us to put out the fire. B. When a strong wind began to blow, it was even more difficult to control the fire. C. If the wind hadn’t been so strong, it would have been much easier to put out the fire. D. As the wind was really very strong, it took them a long time to put out the fire. Question 27: You should have finished the report by now, Jason told his secretary. A. Jason reproached his secretary for not having finished the report. B. Jason said that his secretary had not finished the report. C. Jason reminded his secretary of finishing the report on time. D. Jason scolded his secretary for not having finished the report. Question28: No sooner had Mary begun her new job than she knew she had made a mistake. A. As soon as Mary started working, she realized that her decision had not been a good one. B. Had Mary not just begun a new job, she would have gone looking for a better one. C. Just before Mary took up her new post, she realized that she was not suited for it. D. Since Mary did not like her new job, she began looking for one more suitable to her. Mark the letter on your answer sheet to indicate the sentence that best combines each pair of sentences Question 29: Despite his seeming control over nature, there are still some points where man remains helpless against her. A. Man is defenseless against nature, because he cannot have any influence on her. B. Although man has an apparent control over nature, he hasn't fully conquered her yet. C. Man believes he has complete control over nature, but in many respects, this isn't so. D. Even though man can affect her, in many areas nature still does things better. Question 30: The teacher gave some instructions. I don’t understand any of them. A. The instructions the teacher gave are not understanding to me. B. It was the instructions the teacher gave that confused me. C. It hasn’t been clear to me about the instructions given. D. I’m finding it difficult to figure out what the teacher required according to his instructions Read the following passage and mark the letter A, B, C, or D on your answer sheet to indicate the correct word or phrase that best fits each of the numbered blanks from 31 to 35 Our Homes: Now and Then Everyone needs a home where they feel sheltered and safe. Today we live in modern flats and houses, (31) ___ have air-conditioning to keep us cool, and heating to keep us warm. There is electricity for lighting and supplies of gas or oil for the heating. Hot and cold water (32) ___ from the taps and dirty water disappears (33) ___ the drains. Many of our homes have balconies or gardens. 21
  22. In the past, people made their homes from materials that they found nearby. When we look at different houses we can tell how old they are from the materials used and the way they were built. It was different long (34) ___ people did not have water in their homes and there were no electric lights. To keep warm, they sometimes made (35) ___ inside their homes. With a fire started they could cook their food and heat water. Question 31: A. who B. which C. where D. whose Question 32: A. flows B. finds C. flies D. floats Question 33: A. up B. towards C. on D. down Question 34: A. back B. then C. ago D. time Question 35: A. flames B. food C. fires D. furnaces Read the following passage and mark the letter A, B, C, or D on your answer sheet to indicate the correct answer to each of the questions from 36 to 43 First man on the moon On July 16, 1969, America launched the Apollo 11, Lunar Landing Mission from Kennedy Space Center. This was a 363-foot-tall space vehicle, the five engines of which on the Saturn V rocket generated 7.5 million pounds of thrust. Twelve minutes after the launch, the astronauts were in orbit 120 miles above the Earth. [1] At a speed of 17,400 mph, they began their four-day journey to the moon. [2] They had nearly a quarter of a million miles to go. On July 20, 1969, Neil Armstrong descended from the lunar module ladder. [3] Just prior to taking his first step on the moon, Armstrong pilled on a special ring, causing a TV camera to automatically deploy. As he stepped onto the moon’s surface he proclaimed, “That’s one small step for man, one giant leap for mankind”. [4] However, Armstrong inadvertently omitted the “a” before “man”. This error slightly changed the meaning of what was to become known as Armstrong’s famous statement. Question 36: What is this passage mainly about? A. the first manned mission to the moon B. how fast the rocket traveled C. the first man in space D. the reason Neil Armstrong is so famous Question 37: The word “They” in the passage refers to___. A. Apollo 11 B. astronauts C. Americans D. engines Question 38: The word “deploy” in the passage is closet in meaning to ___. A. photograph B. Begin working C. Stop functioning D. Light up Question 39: Which of the following is the best title for this passage? A. The Life Of Neil Armstrong B. The History Of Space Travel C. The Story Of Apollo 11 D. The Reason Neil Armstrong Is So Famous Question 40: It can be inferred that Armstrong’s statement was important because ___. A. Neil Armstrong was a great speaker B. they were the first words to be spoken on the moon C. this was the first time America had sent people to space D. the statement was spoken from the lunar module ladder Question 41: Which is the best place for the following sentence? “He was about to make history” A. [1] B. [2] C. [3] D. [4] Question 42: The paragraph following the text would most likely discuss ___. A. the events that happened on July 21, 1969 B. the completely successful mission of Apollo 12 C. the pictures that the astronauts took on the Moon D. How the omission of “a” changed the meaning of the statement 22
  23. Question 43: Complete the summary below by choosing one sentence that expresses one of the most important ideas in the passage. Summary: This passage discusses the Apollo 11 space mission. Apollo 11 was an American spacecraft that took the first astronauts to the moon. Armstrong was the first man to step on the moon. ___ A. Apollo 11 was a large vehicle launched by a Saturn V rocket B. The journey to the moon took four days C. Armstrong made a meaningful mistake in his speech as he took his first steps D. A special ring caused a TV camera to begin operating. Read the following passage and mark the letter A, B, C, or D on your answer sheet to indicate the correct answer to each of the questions from 44 to 50 In the West, cartoons are used chiefly to make people laugh. The important feature of all these cartoons is the joke and the element of surprise which is contained. Even though it is very funny, a good cartoon is always based on close observation of a particular feature of life and usually has a serious purpose. Cartoons in the West have been associated with political and social matters for many years. In wartime, for example, they proved to be an excellent way of spreading propaganda. Nowadays cartoons are often used to make short, sharp comments on politics and governments as well as on a variety of social matters. In this way, the modern cartoon has become a very powerful force in influencing people in Europe and the United States. Unlike most American and European cartoons, however, many Chinese cartoon drawings in the past have also attempted to educate people, especially those who could not read and write. Such cartoons about the lives and sayings of great men in China have proved extremely useful in bringing education to illiterate and semi-literate people throughout China. Confucius, Mencius and Laozi have all appeared in very interesting stories presented in the form of cartoons. The cartoons themselves have thus served to illustrate the teachings of the Chinese sages in a very attractive way. In this sense, many Chinese cartoons are different from Western cartoons in so far as they do not depend chiefly on telling jokes. Often, there is nothing to laugh at when you see Chinese cartoons. This is not their primary aim. In addition to commenting on serious political and social matters, Chinese cartoons have aimed at spreading the traditional Chinese thoughts and culture as widely as possible among the people. Today, however, Chinese cartoons have an added part to play in spreading knowledge. They offer a very attractive and useful way of reaching people throughout the world, regardless of the particular country in which they live. Thus, through cartoons, the thoughts and teachings of the old Chinese philosophers and sages can now reach people who live in such countries as Britain, France, America, Japan, Malaysia or Australia and who are unfamiliar with the Chinese culture. Until recently, the transfer of knowledge and culture has been overwhelmingly from the West to the East and not vice versa. By means of cartoons, however, publishing companies in Taiwan, Hong Kong and Singapore are now having success in correcting this imbalance between the East and the West. Cartoons can overcome language barriers in all foreign countries. The vast increase in the popularity of these cartoons serves to illustrate the truth of Confucius’s famous saying “One picture is worth a thousand words.” Question 44: Which of the following clearly characterizes Western cartoons? A. Enjoyment, liveliness, and carefulness B. Seriousness, propaganda, and attractiveness C. Humor, unexpectedness, and criticism D. Originality, freshness, and astonishment Question 45: Chinese cartoons have been useful as an important means of ___. A. educating ordinary people B. spreading Western ideas C. amusing people all the time D. political propaganda in wartime Question 46: The passage is intended to present ___. A. a contrast between Western cartoons and Chinese cartoons B. a description of cartoons of all kinds the world over 23
  24. C. an outline of Western cartoons and Chinese cartoons D. an opinion about how cartoons entertain people Question 47: In general, Chinese cartoons are now aiming at ___. A. bringing education to illiterate and semi-literate people in the world B. disseminating traditional practices in China and throughout the world C. illustrating the truth of Chinese great men’s famous sayings D. spreading the Chinese ideas and cultural values throughout the world Question 48: Which of the following is most likely the traditional subject of Chinese cartoons? A. Jokes and other kinds of humor in political and social matters. B. The philosophies and sayings of ancient Chinese thinkers. C. The illiterate and semi-literate people throughout China. D. The stories and features of the lives of great men the world over. Question 49: According to the passage, which of the following is true? A. Western cartoons always have a serious purpose. B. Language barriers restricted cartoons. C. Cartoons will replace other forms of writing. D. Cartoons can serve various purposes. Question 50: Which of the following could be the best title for the passage? A. Chinese Cartoons and Western Cartoons B. A Very Powerful Force in Influencing People C. An Excellent Way of Spreading Propaganda D. Cartoons as a Way of Educating People TEST 06-12 Mark the letter A, B, C, or D on your answer sheet to indicate the word whose underlined part differs from the other three in pronunciation in each of the following questions. Question 1: A. reserved B. locked C. forced D. touched Question 2: A. arrange B. arise C. area D. arrive Mark the letter A, B, C, or D on your answer sheet to indicate the word that differs from the other three in the position of primary stress in each of the following questions. Question 3: A. household B. secure C. pressure D. active Question 4: A. supportive B. leftovers C. confidence D. hospital Mark the letter A, B, C, or D on your answer sheet to indicate the underlined part that needs correction in each of the following questions Question 5: The police is now investigating the robbery that took place in our neighborhood last night. A B C D Question 6: My pen has very few ink in it, so could you give me some? A B C D Question 7: Neither William or his brother stole the rambutans from our orchard. A B C D Mark the letter A, B, C, or D on your answer sheet to indicate the correct answer to each of the following questions. Question 8: Frankly, I don’t know how he ___on the small amount of money he earns. A. gets down B. gets over C. gets at D. gets by Question 9: ___ my homework last week, I would have got a good mark. A. If I did B. I had done C. Had I done D. Only if I done 24
  25. Question 10: Do you have any objections ___this new road? A. at B. with C. to D for Question 11: Here is the man ___my brother is going to___. A. who his daughter- marry B. whose daughter- get married to C. whose daughter- get married D. his daughter – marry Question 12: He’s always trying ___me. A. to avoid to meet B. avoiding meeting C. to avoid meeting D. avoiding to meet Question 13: Ms. Kent expects ___about any revisions in manuscript before it is printed. A. consulting B. being consulted C. consult D. to be consulted Question 14: After nine months without any rain, the country was facing one of the worst ___in the last thirty years. A. draughts B. floods C. eruptions D. droughts Question 15: If my candidate had won the election, I ___ happy now. A. am B. would be C. can be D. would have been Question 16: George and Paul are the two men ___in my factory. A. working B. to work C. work D. worked Question 17: How much longer do we have to wait? This is starting to get on my___. A. mind B. back C. nerves D. nose Question 18: I enjoy my job as a baker, but it’s taken me over five years to ___to working at night. A. be use B. get used C. used D. getting used Question 19: I find it difficult to pay my bills as prices keep___. A. gaining B. raising C. growing D. rising Mark the letter A, B, C, or D on your answer sheet to indicate the most suitable response to complete each of the following exchanges. Question 20: Lora: “Do you mind if I turn on the fan?” Maria: “ ___.” A. Not for me B. Not at all C. Never mind D. Not enough Question 21: Mary: " Thanks a lot for your help." Nick: "___." A. My happiness B. My delight C. My excitement D. My pleasure Mark the letter A, B, C, or D on your answer sheet to indicate the word(s) CLOSEST in meaning to the underlined word(s) in each of the following questions. Question 22: Once in a while I visit my grandparents on the farm and stay there for some days. A. Regularly B. Sometimes C. Usually D. Rarely Question 23: Within a week on display at the exhibition, the painting was hailed as a masterpiece. A. a large work of art B. an expensive work of art C. an excellent work of art D. a down – to – earth work of art Mark the letter A, B, C, or D on your answer sheet to indicate the word(s) OPPOSITE in meaning to the underlined word(s) in each of the following questions. Question 24: The consequences of the typhoon were disastrous due to the lack of precautionary measures. A. physical B. severe C. beneficial D. damaging Question 25: Vietnam’s admission to the Word Trade Organization (WTO) has promoted its trade relations with other countries. A. balanced B. restricted C. expanded D. boosted Mark the letter A, B, C, or D on your answer sheet to indicate the sentence that is closest in meaning to each of the following questions. Question 26: "Why don't you get up early to do morning exercises?" Peter asked me. A. Peter advised me to get up early to do morning exercises. 25
  26. B. Peter recommended me not to get up early to do morning exercises. C. Peter told me the reason why I did not get up early to do morning exercises. D. Peter suggested that he should get up early to do morning exercises. Question 27: Alan worked too hard at the office, and this led to his illness. A. Because Alan worked too hard at the office, and this led to his illness. B. Alan's working too hard at the office, and this led to his illness. C. Alan's illness resulted from his working too hard at the office. D. His working too hard at the office resulted from his illness. Question 28: It was overeating that caused his heart attack. A. But for his overeating, he wouldn't have had a heart attack. B. But for his overeating, he wouldn't have a heart attack. C. If it wasn't his overeating, he wouldn't have had a heart attack. D. Had he had overeating, he wouldn't have had a heart attack. Read the following passage and mark the letter A, B, C, or D on your answer sheet to indicate the sentence that best combines each pair of sentences in the following questions. Question 29: They booked the hotel. They had stayed there on their honeymoon. A. They booked the hotel where they had stayed on their honeymoon. B. They booked the hotel where they had stayed there on their honeymoon. C. They booked the hotel which they had stayed on their honeymoon. D. They booked the hotel which they had stayed there on their honeymoon. Question 30: Anne jogs every morning. It is very good for her health. A. Anne jogs every morning and is very good for her health. B. Anne jogs every morning, which is very good for her health. C. Anne jogs every morning and then it is very good for her health. D. Anne jogs every morning that it is very good for her health Read the following passage and mark the letter A, B, C, or D on your answer sheet to indicate the correct word or phrase that best fits each of the numbered blanks from 31 to 35. Why is it that many teenagers have the energy to play computer games until late at night but can’t find the energy to get out of bed in time for school? According to a new report, today’s generation of children are in danger of getting so (31) sleep that they are putting their mental and physical health at risk. Adults can easily survive on seven to eight hours’ sleep a night, (32) teenagers require nine or ten hours. According to medical experts, one in five youngsters gets anything between two and five hours’ sleep a night less than their parents did at their age. This (33) serious questions about whether lack of sleep is affecting children’s ability to concentrate at school. The concentration between sleep deprivation and lapses in memory, impaired reaction time and poor concentration is well established. Research has shown that losing as little as half an hour’s sleep a night can have profound effects (34) how children perform the next day. A good night’s sleep is also crucial for teenagers because it is while they are asleep that they release a hormone that is essential for their “growth spurt” (the period during teenage years when the body grows at a rapid rate). It’s true that they can, to some (35) , catch up on sleep at weekends, but that won’t help them when they are dropping off to sleep in class on a Friday afternoon. Question 31: A. less B. little C. few D. much Question 32: A. because B. so C. or D. whereas Question 33: A. rises B. raises C. comes D. results Question 34: A. on B. in C. at D. to Question 35: A. level B. rate C. point D. extent Read the following passage and mark the letter A, B, C, or D on your answer sheet to indicate the correct answer to each of the questions from 36 to 42. 26
  27. The goal of Internet-based encyclopedia Wikipedia (www.wikipedia.org) is to give everyone on the planet access to information. Like other encyclopedias, Wikipedia contains lots of information: more than 2.5 million articles in 200 different languages covering just about every subject. Unlike other encyclopedias, however, Wikipedia is not written by experts, but by ordinary people. These writers are not paid and their names are not published. They contribute to Wikipedia simply because they want to share their knowledge. Encyclopedias began in ancient times as collections of writings about all aspects of human knowledge. The word itself comes from ancient Greek, and means “a complete general education”. Real popularity for encyclopedias came in the nineteenth century in Europe and the United States, with the publication of encyclopedias written for ordinary readers. With the invention of the CD-ROM, the same amount of information could be put on a few computer discs. Then with the Internet, it became possible to create an online encyclopedia that could be constantly updated, like Microsoft’s Encarta. However, even Internet- based encyclopedias like Encarta were written by paid experts. At first, Wikipedia, the brainchild of Jimmy Wales, a businessman in Chicago, was not so different from these. In 2001, he had the idea for an Internet-based encyclopedia that would provide information quickly and easily to everyone. Furthermore, that information would be available free, unlike other Internet encyclopedias at that time. But Wales, like everyone else, believed that people with special knowledge were needed to write the articles, and so he began by hiring experts. He soon changed his approach, however, as it took them a long time to finish their work. He decided to open up the encyclopedia in a radical new way, so that everyone would have access not only to the information, but also to the process of putting this information online. To do this, he used what is known as “Wiki” software (from the Hawaiian word for “fast”), which allows users to create or alter content on web page. The system is very simple: When you open the web site, you can simply search for information or you can log on to become a writer or editor of articles. If you find an article that interests you – about your hometown, for example – you can correct it or expand it. This process goes on until no one is interested in making any more changes. Question 36: Wikipedia is written by___. A. paid written B. millionaires C. normal people D. world experts Question 37: The phrase “these writers” in the first paragraph refers to ___. A. ordinary readers B. ordinary people C. encyclopedia experts D. every subject Question 38: The word “brainchild” in the second paragraph of the passage can be best replaced by___. A. child B. father C. born D. product Question 39: The word “approach” in the third paragraph of the passage means ___. A. method B. idea C. writing D. time Question 40: The user of Wikipedia can do all of the followings EXCEPT ___. A. have access to information B. edit information C. modify information D. determinate the website Question 41: We can say that Jimmy Wales ___. A. became very famous after the formation of Wikipedia B. made a great profit from Wikipedia C. is the father of Wikipedia D. decides who can use Wikipedia Question 42: Wiki software enables ___. A. editing C. a purchase of information B. limited access D. exchanging articles Read the following passage and mark the letter A, B, C, or D on your answer sheet to indicate the correct answer to each of the questions from 43 to 50. It is commonly believed that school is where people go to get an education. Nevertheless, it has been said that today children interrupt their education to go to school. The difference between schooling and education implied by this remark is important. Education is much more open-ended and all-inclusive than schooling. Education knows no limits. It can take place anywhere, whether in the shower or on the job, whether in the kitchen or on a tractor. It includes both the formal learning that takes place in school and the whole universe of informal learning. The agent (doer) of education can vary from respected grandparents to the people arguing about politics on the radio, from a child to a famous scientist. Whereas schooling has a certain 27
  28. predictability, education quite often produces surprises. A chance conversation with a stranger may lead a person to discover how little is known of other religions. People receive education from infancy on. Education, then, is very broad, inclusive term; it is a lifelong process, a process that starts long before the start of school, and one that should be a necessary part of one’s entire life. Schooling, on the other hand, is a specific, formalized process, whose general pattern varies little from one setting to the next. Throughout a country, children arrive at school at about the same time, take the assigned seats, are taught by an adult, use similar textbooks, do homework, take exams, and so on. The pieces of reality that are to be learned, whether they are the alphabet or an understanding of the workings of governments, have been limited by the subjects being taught. For example, high school students know that they are not likely to find out in their classes the truth about political problems in their society or what the newest filmmakers are experimenting with. There are clear and undoubted conditions surrounding the formalized process of schooling. Question 43: This passage is mainly aimed at___. A. giving examples of different schools B. telling the difference between the meaning of two related words C. listing and discussing several educational problems D. telling the story about excellent teachers Question 44: In the passage, the expression ”children interrupt their education to go to school” mostly implies that___. A. education is totally ruined by schooling B. schooling prevents people discovering things C. all of life is an education D. schooling takes place everywhere Question 45: The word “all-inclusive “ in the passage mostly means ___. A. allowing no exceptions B. including everything or everyone C. involving many school subjects D. going to many directions Question 46: According to the passage, the doers of education are___. A. almost all people C. mainly politicians B. only respected grandparents D. mostly famous scientists Question 47: Which of the followings would the writer support? A. Schooling is of no use because students do similar things every day. B. Our education systems needs to be changed as soon as possible. C. Without formal education, people won’t be able to read and write. D. Going to school is only part of how people become educated. Question 48: The word ” they” in the last paragraph refers to ___. A. newest filmmakers B. high school students C. workings of governments D. political problems Question 49: Because the general pattern of schooling varies little from one setting to the text, school children throughout the country___. A. are taught by the same teachers B. have the same abilities C. have similar study conditions D. do similar things Question 50: Which of the followings is TRUE according to the passage? A. The best schools teach a variety of subjects. B. The more years students go to school, the better their education is. C. Students benefit from schools, which require long hours and homework. D. Education and schooling are quite different experience. ___THE END___ TEST 07-13 Mark the letter A, B, C, or D on your answer sheet to indicate the word whose underlined part is pronounced differently from that of the rest in each of the following questions. Question 1: A. wicked B. watched C. stopped D. cooked Question 2: A. head B. bread C. clean D. lead 28
  29. Mark the letter A, B, C or D on your answer sheet to indicate the word that differs from the other three in the position of primary stress in each of the following questions. Question 3: A. familiarB. impatient C. uncertain D. arrogant Question 4: A. disappear B. arrangement C. opponent D. contractual Mark the letter A, B, C, or D on your answer sheet to indicate the correct answer to each of the following questions. Question 5: There is too much in this world. A. greediness B. greed C. greedy D. greedness Question 6: - "That's a very nice skirt you're wearing." - “ ” A. How a compliment! B. That's all right. C. It's nice of you to say so. D. I like you said so. Question 7: are the formal rules of correct or polite behavior among people using the Internet. A. Traffic rules B. Family rules C. Codes of etiquettes D. Codes of netiquettes Question 8: - " " - "Yes, of course. " A. You won't help me this time. B. You'd better give me one hand. C. I don't think I'll need your help. D. Could you give me a hand? Question 9: UNICEF supports and funds for the most disadvantaged children all over the world. A. presents B. assists C. provides D. offers Question 10: He would win the race if he his brother's example and trained harder. A. repeated B. set C. answered D. followed Question 11: "Excuse me. Where is the office of OXFAM located?" A. leading B. head C. central D. summit Question 12: The doctors know that it is very difficult to save the patient's life, they will try their best. A. but B. although C. despite D. however Question 13: I am sending you my curriculum vitae you will have a chance to study it before our interview. A. so that B. because C. for D. since Question 14: Everybody in the house woke up when the burglar alarm . A. went out B. went off C. came about D. rang off Question 15: Have a piece of chocolate, ? A. do you B. would you C. don't you D. haven't you Question 16: By the time you come here tomorrow, the work . A. will have been finishing B. will be finishing C. will have been finished D. will be finished Question 17: If you put your money in a bank now, you may get 12%___annually. A. interest B. profit C. money D. income Question 18: I can’t possibly lend you any more money, it is quite out of the . A. order B. practice C. place D. question Mark the letter A, B, C, or D on your answer sheet to indicate the word(s) CLOSEST in meaning to the underlined word(s) in each of the following questions. Question 19: Such problems as haste and inexperience are a universal feature of youth. A. marked B. separated C. shared D. hidden 29
  30. Question 20: We have lived there for years and grown fond of the surroundings. That is why we do not want to leave. A. loved the surroundings B. planted many trees in the surroundings C. possessed by the surroundings D. haunted by the surroundings Mark the letter A, B C or D on your answer sheet to indicate the word(s) OPPOSITE in meaning to the underlined word(s) in each of the following questions. Question 21: He is a typical optimist, always looking on the bright side of everything. A. pessimist B. introvert C. extrovert D. activist Question 22: When I was going shopping yesterday, I accidentally met one of my old friends in high school. A. by far B. by heart C. by chance D. on purpose Read the following passage and mark the letter A, B, C or D on your answer sheet to indicate the correct answer for each of the blanks from 30 to 39. The universal symbol of the Internet era communications, the @ sign used in e-mail addresses to signify the word 'at', is (23) a 500-year-old invention of Italian merchants, a Rome academic has revealed. Giorgio Stabile, a science professor at La Sapienza University, claims to have stumbled on the earliest known example of the symbol's use, as a(n) (24) of a measure of weight or volume. He says the sign represents an amphora, a measure of capacity based on the terracotta jars used to transport grain and liquid in the ancient Mediterranean world. The professor unearthed toe ancient symbol in the course of research for a visual history of the 20th century, to be published by the Treccani Encyclopedia. The first (25) instance of its use, he says, occurred in a letter written by a Florentine merchant on May 4, 1536. He says the sign made its way along trade routes to northern Europe, where it came to represent 'at the price of’, its contemporary accountancy meaning. Professor Stabile believes that Italian banks may possess even earlier documents (26) the symbol lying forgotten in their archives. The oldest example could be of great value. It could be used (27) publicity purposes and to enhance the prestige of the institution that owned it, he says. The race is on between the mercantile world and the banking world to see who has the oldest documentation of @. Question 23: A. actually B. truly C. essentially D. accurately Question 24: A. proof B. sign C. evidence D. indication Question 25: A. known B. knowing C. knowable D. knowledgeable Question 26: A. taking B. carrying C. delivering D. bearing Question 27: A. on B. for C. with D. by Read the following passage and mark the letter A, B, C, or D on your answer sheet to indicate the correct answer to each of the questions from 40 to 49. A rather surprising geographical feature of Antarctica is that a huge freshwater lake, one of the world's largest and deepest, lies hidden there under four kilometers of ice. Now known as Lake Vostok, this huge body of water is located under the ice block that comprises Antarctica. The lake is able to exist in its unfrozen state beneath this block of ice because its waters are warmed by geothermal heat from the earth's core. The thick glacier above Lake Vostok actually insulates it from the frigid temperatures on the surface. The lake was first discovered in the 1970s while a research team was conducting an aerial survey of the area. Radio waves from the survey equipment penetrated the ice and revealed a body of water of indeterminate size. It was not until much more recently that data collected by satellite made scientists aware of the tremendous size of the lake; the satellite-borne radar detected an extremely flat region where the ice remains level because it is floating on the water of the lake. 30
  31. The discovery of such a huge freshwater lake trapped under Antarctica is of interest to the scientific community because of the potential that the lake contains ancient microbes that have survived for thousands upon thousands of years, unaffected by factors such as nuclear fallout and elevated ultraviolet light that have affected organisms in more exposed areas. The downside of the discovery, however, lies in the difficulty of conducting research on the lake in such a harsh climate and in the problems associated with obtaining uncontaminated samples from the lake without actually exposing the lake to contamination. Scientists are looking for possible ways to accomplish this. Question 28: What is true of Lake Vostok? A. It is completely frozen. B. It is a saltwater lake. C. It is beneath a thick slab of ice. D. It is heated by the sun. Question 29: All of the following are true about the 1970 survey of Antarctica EXCEPT that it . A. was conducted by air B. made use of radio waves C. could not determine the lake's exact size D. was controlled by a satellite Question 30: It can be inferred from the passage that the ice would not be flat if . A. there were no lake underneath B. the lake were not so big C. Antarctica were not so cold D. radio waves were not used Question 31: The word "microbes" in paragraph 3 could best be replaced by which of the following? A. Pieces of dust B. Tiny bubbles C. Tiny organisms D. Rays of light Question 32: Lake Vostok is potentially important to scientists because it A. can be studied using radio waves B. may contain uncontaminated microbes C. may have elevated levels of ultraviolet light D. has already been contaminated Question 33: The last paragraph suggests that scientists should be aware of A. further discoveries on the surface of Antarctica B. problems with satellite-borne radar equipment C. ways to study Lake Vostok without contaminating it D. the harsh climate of Antarctica Question 34: The purpose of the passage is to . A. explain how Lake Vostok was discovered B. provide satellite data concerning Antarctica C. discuss future plans for Lake Vostok D. present an unexpected aspect of Antarctica's geography Read the following passage and mark the letter A, B, C, or D on your answer sheet to indicate the correct answer to each of the questions from 50 to 59. Perhaps the most striking quality of satiric literature is its freshness and its originality of perspective. Satire itself, however, rarely offers original ideas. Instead, it presents the familiar in a new form. Satirists do not offer the world new philosophies. What they do is look at familiar conditions from a perspective that makes these conditions seem foolish, harmful, or affected. Satire jars us out of complacence into a pleasantly shocked realization that many of the values we unquestioningly accept are false. Don Quixote makes chivalry seem absurd; Brave New World ridicules the pretensions of science; A Modest Proposal dramatizes starvation by advocating cannibalism. None of these ideas is original. Chivalry was suspect before Cervantes, humanists objected to the claims of pure science before Aldous Huxley, and people were aware of famine before Swift. It was not the originality of the idea that made these satires popular. It was the manner of expression, the satiric method, that made them interesting and entertaining. Satires are read because they are aesthetically satisfying works of art, not because they are morally wholesome or ethically instructive. They are stimulating and refreshing because with commonsense briskness they brush away illusions and secondhand opinions. With spontaneous irreverence, satire rearranges perspectives, scrambles familiar objects into incongruous juxtaposition, and speaks in a personal idiom instead of abstract platitude. Satire exists because there is need for it. It has lived because readers appreciate a refreshing stimulus, an irreverent reminder that they live in a world of platitudinous thinking, cheap moralizing, and foolish philosophy. Satire serves to prod people into an awareness of truth, though rarely to any action on 31